Je veux une intégrale !

Bonjour
Je veux une intégrale à calculer pour cette fin de semaine . Une proposition ?
Merci.
Le 😄 Farceur


«1345

Réponses

  • Pour $a,b,u,v,u+v-1>0$ calculer $\int_{-\infty}^{\infty}\frac{dt}{(a-it)^u(b+it)^{v}}$
  • Merci cher P
    Ca va m' occuper
    Le 😄 Farceur


  • Je peux te proposer:
    \begin{align}\int_0^\infty \left(\frac{\arctan x\ln x}{x}\right)^2\,dx\end{align}
  • Merci, un vrai régal.
    Le 😄 Farceur


  • Et celle-ci :
    $$\int_ 1 ^2 \frac{(x-1)\sqrt{x^2 + 2x + 5}}{x^2 +1}$$
    Maple la calcule en deux secondes, mais j'ignore si un humain serait capable de trouver sa valeur.
  • En posant $x=2\sinh t -1$ on se ramene a une fraction rationnelle en $e^t:$ voir cours de Dixmier, mathematiques premiere annee des universities, chapitre primitives se ramenant a celles de fractions rationnelles, type IV.
  • Encore une ?
    $$\int_0^\infty \frac{e^{-x^2}-1+x^2}{x^4} \, \textrm{d}x.$$
  • Mh, très intéressant, je vais regarder ce samedi.
    Le 😄 Farceur


  • La dernière intégrale, me semble-t-il, peut se calculer par deux intégrations par parties successives et on se retrouve à évaluer:
    \begin{align}\int_0^\infty \text{e}^{-x^2}\,dx\end{align}
  • J'utilise $e^{-u}=1-u+\int_{0}^u(u-t)e^{-t}.$
  • P. Cela te permet d'éviter d'avoir à calculer $$

    \int_0^\infty \text{e}^{-x^2}\,dx\qquad ?
    $$
  • Non bien sur.
  • On peut éviter le recours à cette intégrale B-)-
  • Noix de Totos:
    Mais j'imagine qu'il faut utiliser un contour et le théorème des résidus.
  • Non, mais on peut "tricher" si on connaît un résultat sur la fonction Gamma.
  • Bonjour,

    Pour calculer $\displaystyle \int_{0}^{+\infty} {e^{-x^2}-1+x^2\over x^4} dx$ on montre l’existence par un équivalent de l’intégrande en $0$, puis on considère $\displaystyle I(a)=\int_{0}^{+\infty} {e^{-a x^2}-1+a x^2\over x^4} dx$ pour $a>0$, on a $\displaystyle I’(a)=\int_{0}^{+\infty} {-x^2 e^{-a x^2}+x^2\over x^4} dx$ puis $\displaystyle I^{(2)} (a)=\int_{0}^{+\infty} e^{-a x^2} dx=k/\sqrt{a}$, qu’on intègre en $\displaystyle I’(a)=2 k \sqrt{a}$ puisque $I’(0)=0$, qu’on intègre en $\displaystyle I(a)=2/3 .a^{3/2} .2k$ et on conclut $I=I(1)=2/3 .2k$ avec le résultat classique $k=\sqrt{\pi}/2.$
  • Je voulais attendre samedi, puisque Gebrane avait mentionné ce délai, mais puisqu'une solution a été donnée ci-dessus par YvesM, en voilà une autre possible, qui utilise l'outil suivant auquel je faisais allusion.

    Lemme (Cauchy & Saalschütz). Soit $n \in \mathbb{Z}_{\geqslant 0}$ et $z \in \mathbb{C}$ tel que $-n-1 < \textrm{Re}(z) < -n$. Alors
    $$\int_0^\infty t^{z-1} \left( e^{-t} - \sum_{k=0}^n \frac{(-1)^k}{k!} \right) \, \textrm{d}t = \Gamma(z).$$

    Référence.

    E. T. Whittaker & G. N. Watson, A Course of Modern Analysis, Cambridge, p. 243.

    On change alors de variable $t=x^2$ et on applique ce résultat avec $n=1$ et $z = - \frac{3}{2}$
    $$\int_0^\infty \frac{e^{-x^2}-1+x^2}{x^4} \, \textrm{d}x = \frac{1}{2} \int_0^\infty t^{-5/2} \left( e^{-t} + 1 - t \right) \, \textrm{d}t = \frac{1}{2} \Gamma \left( - \frac{3}{2} \right) = \frac{2}{3} \Gamma \left( \frac{1}{2} \right) = \frac{2 \sqrt{\pi}}{3}.$$
  • Merci Noix de Totos pour ce lemme que je ne connaissais pas.
    Je vois souvent en référence ce cours d'analyse il semble receler des pépites.
  • Je suis un peu perdu (première lecture de ce fil ce samedi)
    Vous m'avez laissé quoi ?
    Qui a mangé tout le gâteau ?
    Le 😄 Farceur


  • Gebrane:

    Il te reste:
    http://www.les-mathematiques.net/phorum/read.php?4,1830862,1830962#msg-1830962

    Personne ne s'y est attaqué, de l'arctangente mâtiné de logarithme au carré, cela en a effrayé plus d'un. B-)-
  • Bah c'est trop facile lorsque on connait non ? :-D
    Le 😄 Farceur


  • Pour ceux qui ne connaissent pas mes blagues, cliquez sous le visage souriant du message plus haut
    Le 😄 Farceur


  • Gebrane:

    Je n'ai pas encore essayé de calculer cette intégrale. J'ai seulement vu une solution qui traînait quelque part mais qui ne me plait pas beaucoup.
  • FDP

    Ne trouves-tu pas à ton gout la solution de Felix Marin, elle est trops belle et s'appuie sur $$\int_{0}^{1}{d y \over x^{2}y^{2} + 1}
    ={\arctan x \over x}$$
    Le 😄 Farceur


  • Je vous souhaite une bonne fin de semaine, ils m'ont rien laissé
    je vais passer ma journée en montagne
    a+
    Le 😄 Farceur


  • Gebrane:

    Je ne savais pas qu'il y avait une solution sur M.E (ou j'ai oublié).
    La solution que j'ai vue ailleurs utilisait le théorème des résidus.
  • Allez, une autre...Valeur exacte de
    $$I = \int_1^2 \frac{\textrm{d}x}{1+x^3}.$$
  • L'intégrande admet une primitive qui a une expression "simple".
  • Comment faites-vous pour la première intégrale de FdP (à savoir $\displaystyle \int_0^{+\infty} \left(\dfrac{\arctan(x)\ln(x)}{x}\right)^2 dx$). Je tourne en rond, et ni maple ni Wolfram Alpha n'y arrive.
  • Bonjour,

    $\int_0^1 {dt \over 1 +x^2 t^2}$ et intégrale triple et intégrales classiques.
  • @FdP : vraiment "simple" ? Moi, j'ai utilisé les résidus, car tes primitives "simples", sans logiciel...
  • On peut commencer par une intégration par parties.
  • Noix de Totos:

    On vérifie facilement qu'une fonction est une primitive si on vous donne cette primitive.

    Primitive pas si difficile à trouver me semble-t-il:

    $1+x^3=(1+x)(1-x+x^2)$

    $\dfrac{1}{1+x^3}=\dfrac{1}{3(1+x)}-\dfrac{x}{3(1-x+x^2)}+\dfrac{2}{3(1-x+x^2)}$
  • Oui, suis-je bête, j'étais fixé sur les résidus...Bon aucun intérêt.

    J'en remets donc une autre (j'espère que celle-ci sera intéressante) :
    $$I = \int_0^{2 \pi} \frac{x}{2+\cos x} \, \textrm{d}x.$$
  • Bonjour,

    Il fait trop chaud :
    - existence évidente,
    - CDV $y=2 \pi -x$, $2 I=2 \pi \int_{[0,2 \pi]} {dx\over 2+\cos x}$,
    - intégrande pair donc intégration sur $[-\pi,+\pi]$ et formation d’un cosinus au dénominateur... et intégration par arctangente,
    - $I=2/\sqrt{3}.\pi^2.$
  • Bon, après un bon paquets de calculs, j'ai fini par trouver. Je poste le résultat ici pour en garder une trace :
    \[ \int_0^{+\infty} \left(\dfrac{\arctan(x)\ln(x)}{x}\right)^2 dx = \dfrac{1}{4}\pi^3 \ln(2) - \dfrac{\pi^3}{12} + 2 \pi \ln(2) - \dfrac{1}{4} \pi \zeta(3) \]
  • YvesM est le (ou l'un des) champion(s) du calcul intégral de ce forum ! Bravo !

    On pouvait aussi utiliser la méthode des résidus qui, accessoirement, permet également, et en même temps, d'obtenir la valeur de l'intégrale "duale"
    $$\int_0^\infty \frac{\textrm{d}x}{2 + \textrm{ch} \, x}.$$
  • @Guego
    Ta méthode aussi passe par une intégrale triple?
    edit @noix de toto
    Il faut augmenter la dose pour pouvoir freiner YvesM ou FDP
    Le 😄 Farceur


  • gebrane a écrit:
    Il faut augmenter la dose pour pouvoir freiner YvesM ou FDP.

    Bon ! Et celle-ci :
    $$\int_0^{\pi/2} e^{\cos x} \cos \left( x + \sin x \right) \, \textrm{d}x \ ?$$
  • Bonjour,

    Effectivement cette dernière me sèche un mauvais jour, mais pas aujourd’hui :
    Une primitive est de la forme $\exp(\cos x) f(x)$ et on dérive, puis on identifie à l’intégrande pour lequel on utilise l’identité $\cos(a+b)=\cos a\cos b-\sin a\sin b$ : $\sin(1).$ Voilà !
  • Il suffit d'adapter la preuve de:

    https://math.stackexchange.com/questions/3278391/int-0-pi-e-cos-x-cosx-sin-x-dx

    En faisant probablement le changement de variable $y=4x$ préalablement par exemple.

    Mais trouver une preuve sans théorème des résidus serait intéressant.
  • Est-ce qu'on peut trouver une primitive de la fonction $F(x)=e^{\cos x} \cos \left( x - \sin x \right)$?
  • Bien !

    J'ai fait comme ceci :

    $$ \int_0^{\pi/2} e^{\cos x} \cos(x+\sin x) \, \textrm{d}x = \textrm{Re} \ \int_0^{\pi/2} e^{ix} \exp \left(e^{ix} \right) \, \textrm{d}x = \textrm{Re} \ \left( \left[-i \exp \left(e^{ix} \right) \right]_0^{\pi/2} \right) = \sin(1).$$
  • Mon stock d'intégrales commence à s'épuiser, mais il me reste celle-ci : $$

    \int_0^\infty e^{-x^2} \, \textrm{ch} \, \big( (2y+1) x \big) \, \textrm{d}x ,\quad \left(y \in \mathbb{R} \right).$$
  • Ho, gaussienne!
  • Je rejoins P, c'est facile
    Le 😄 Farceur


  • Bon !

    Si $\psi(x) = x - \lfloor x \rfloor - \frac{1}{2}$ désigne la première fonction de Bernoulli, déterminer
    $$I = \int_1^\infty \frac{\psi(x)}{x^2} \, \textrm{d}x \quad \textrm{et} \quad J = \int_1^\infty \frac{\psi(x) \log x}{x^2} \, \textrm{d}x.$$
  • pour la I c'est classque $\zeta(s) = s\int_1^\infty \frac{\lfloor x \rfloor - x + 1/2}{x^{s+1}} \mathrm{d}x + \frac1{1-s} + \frac12$

    edit 1 Non je raconte n'importe quoi, le s de noix de toto est egale à 1
    edit 2 le I me semble diverge
    Le 😄 Farceur


  • Non, $I$ et $J$ convergent puisque $\left| \psi(x) \right| \leqslant \frac{1}{2}$. Ainsi, $\max \left( \left|I \right| , \left| J \right| \right) \leqslant \frac{1}{2}$.
  • $I$ est classique et s'exprime avec $\gamma$.

    $J$ l'est moins, elle s'exprime avec $\gamma$ et $\gamma_1$ (constante de Stieltjes).
Connectez-vous ou Inscrivez-vous pour répondre.